What is a number that when you divide it by 2 and subtract 3.8 from the quotient, you get 7?

Answers

Answer 1

Let the number x.

Then the number divided by 2 gives

[tex]\frac{x}{2}[/tex]

Subtract 3.8 from the quotient gives

[tex]\frac{x}{2}-3.8[/tex]

Hence,

[tex]\frac{x}{2}-3.8=7[/tex][tex]\begin{gathered} \frac{x}{2}-3.8=7 \\ \frac{x}{2}=7+3.8=10.8 \\ \Rightarrow x=2\times10.8=21.6 \end{gathered}[/tex]

x = 21.6


Related Questions

Evaluate the expression a-5 -b, when a= 10 and b=4

Answers

Given the expression:

a - 5 - b

we want to evaluate it when a = 10 and b = 4. To do it, we have to replace these values into the equation as follows:

10 - 5 - 4 = 1

A boat heading out to sea starts out at Point AA, at a horizontal distance of 1035 feet from a lighthouse/the shore. From that point, the boat’s crew measures the angle of elevation to the lighthouse’s beacon-light from that point to be 8^{\circ} ∘ . At some later time, the crew measures the angle of elevation from point BB to be 5^{\circ} ∘ . Find the distance from point AA to point BB. Round your answer to the nearest foot if necessary.

Answers

The angle of elevation reduces. The distance between points A and B is roughly 1,269.9 ft.

The distance between point A and the lighthouse is 968 feet.

The angle of elevation from point A to the lighthouse is 7 degrees.

Elevation angle from point B to the lighthouse = 3°

The distance between points A and B.

tan 7° = Height of the sea /  968 feet

tan(7°) 968 feet 118.86 feet = height to the lighthouse

We have from point B;

tan 3° = 118.86 / Horizontal distance from B to HL

Therefore;

Horizontal distance from LH to boat = 118.6 / tan 30° = 2237.9 = feet

Therefore;

D is the distance between A and B.

D = Horizontal distance between B and LH - Horizontal distance between A and LH

A to B distance = 2237.9 feet − 968 feet 1269.9 feet

The distance between points A and B is 1,269.9 ft.

Learn more about trigonometric ratios at

https://brainly.com/question/25909273

#SPJ9

Answer:

The Boat traveled 628 ft, from point A to point B.

Step-by-step explanation:


Its 628 bc I just got it correct on my hw.

can anyone solve? 14 x 3 - 2 + 3

Answers

Answer:

43

Step-by-step explanation:

So, the key to solving this is PEMDAS, or parenthesis, exponents, multiplication/division from left to right, and addition/subtraction from left to right. So, you start off with 14x3, which is 42. So, you have 42-2+3. Next, you have to do 42-2, which is 40. Finally, you do 40+3, which is 43.

Answer:

43

Step-by-step explanation:

14 x 3 - 2 + 3

PEMDAS says multiply before adding or subtracting

42 - 2 + 3

Then subtract

40 +3

Then add

43

Four points W, X, Y, Z are chosen at random on the circumference of a circle. What is the probability that WX > YZ?

Answers

The probability to choose WX > YZ is 1/2.

What is mean by Probability?

The term probability refers to the likelihood of an event occurring.

Given that;

Four points W, X, Y, Z are chosen at random on the circumference of a circle.

Now,

By the four points W, X, Y, Z we make two line WX and YZ.

So, There are two relation between the line WX and YZ.

Thus,  The probability to choose WX > YZ is;

Probability = Possible event / Total possibility

                 = 1/2

Therefore, The probability to choose WX > YZ is 1/2.

Learn more about the probability visit:

https://brainly.com/question/24756209

#SPJ1

Please help !
Place the following real numbers in order from least to greatest.


227

-3

-4.5

165

Answers

Answer: -4.5,-3,165,227

Step-by-step explanation:

What is the solution set to -3x - 39 > 12

Answers

Answer:

x < -17

Step-by-step explanation:

-3x - 39 > 12  Add 39 to both sides

-3x > 51  Divide by -3 and flip the inequality sign

x<-17

Answer: x < -17

Step-by-step explanation: add 39 to both sides then divide 3 on 3x and 51 That is how you answer it. flip the sign to a positive

Jordan is constructing a aregular hexagon inscribed in a circle. He labels the points on the circle as shown

Answers

Given,

The diagram of the circle with the vertex of the hexgon.

Required

The measure of arc VX, angle ZXV.

As known that,

The measure of the circle is 360 degree.

The measure of arc VX is,

[tex]Arc\text{ VX = 60 degree.}[/tex]

The measure of angle ZXV is,

[tex]\angle ZXV=60^{\circ}[/tex]

The measure of arc WZ is 180 degree.

Answer:

VX=120

ZXV=60

WZ

Step-by-step explanation: YOUR WELCOME PUTAS

What is the value of the function at x=−3? Responses y = 3 y, = 3 y = 0 y, = 0 y = 4 y, = 4 y=−3

Answers

The function has a value of y = 4 when x =-3

How to determine the value of the function?

The graph that completes the question is added as an attachment

From the graph, we can see that:

The graph of the function is a linear function

This is so because the function is a straight line

Note that:

Linear equations are equations that have constant average rates of change.

On the straight line, we can see that

The value of the function when x = -3 is 4

This means that

y = 4

Hence, the value of the function at x = -3 is y = 4

Read more about functions at

https://brainly.com/question/5994384

#SPJ1

How can you solve, write and graph inequalities on a number line?

Answers

Let's use the following inequation as an example:

[tex]2y>6[/tex]

First let's simplify it dividing the inequation by 2:

[tex]y>3[/tex]

Now, let's draw a number line, and include the point 3 in it:

We want the values of y that are greater than 3, that is, the numbers to the right of the number 3 in the number line. So we mark the area that corresponds to the values of y that satisfies the inequation:

The number 3 is not part of the answer, because 3 is not greater than 3, so we draw a small empty circle on the number 3. If the number 3 was part of the answer, the circle would be filled.

A recipe requires 2 and 1 third cups of flower the chef needs to increase the recipe bya factor of 4 how many cups of flower does the chef need

Answers

The number of cups needed by the chef is 9 1/3 cups.

How to calculate the fraction?

A fraction simply means a part of a whole number.

In this case, the recipe requires 2 and 1 third cups of flower the chef needs to increase the recipe by a factor of 4.

The cups needed will be:

= 2 1/3 × 4

= 7/3 × 4

= 28/3

= 9 1/3 cups.

This illustrates the concept of fractions.

Learn more about fractions on:

brainly.com/question/17220365

#SPJ1

I only need the answer
THANK YOU!!

Answers

The expression is √3/2 (sin x) - (cos x)/2

Here we have

sin( x + 11π/6)

We have to express it in terms of sin(x) and cos(x)

sin ( 2π - π/6 + x)

sin ( 2π + ( x - π/6))

sin(2π + Ф ) = sin Ф

So

sin( x - π/ 6)

Formula :

sin(A -B) = sin A cos B - cos A sin B

sin( x - π/ 6)

sin x cos π/6 - cos x sin π/6

con π/6 = √3/2  and sin π/6 = 1/2

√3/2 sin x - 1/2 cos x

Therefore the expression is √3/2 sin x - 1/2 cos x.

To know more about trigonometry refer to the link given below:

https://brainly.com/question/10083069

#SPJ1

What is the measure of angle 1 in the picture below?2135°O 90 degreesO 225 degreesO 45 degreesO 135 degrees

Answers

1 and 135 are corresponding angles.

Corresponding angles on parallel lines are equal

m<1= 135 degrees

Find the product and write your answer in proper scientific notation (4.5 x 10³)/(3 x 10⁶)

Answers

Answer:

[tex]1.5\times10^{-3}[/tex]

Explanation:

Given the rational expression

[tex]\frac{4.5\times10^3}{3\times10^6}[/tex]

We can rewrite the fraction as:

[tex]=\frac{4.5}{3}\times\frac{10^3}{10^6}[/tex]

We then simplify and apply the division law of indices to obtain:

[tex]\begin{gathered} =1.5\times10^{3-6} \\ =1.5\times10^{-3} \end{gathered}[/tex]

Note: When you have the same base of an exponent and the terms are being divided, we simply subtract the power of the numerator from that of the denominator.

how would i rewrite the polynomial in the form ax^2+bx+c and then identify the values of a,b, and c1/3x^2+7-6x

Answers

Answer:

a=1/3, b=-6, c=7

Explanation:

Given the polynomial:

[tex]\frac{1}{3}x^2+7-6x​[/tex]

We reorder it to rewrite the polynomial in the form ax²+bx+c:

[tex]\frac{1}{3}x^2-6x​+7[/tex]

Next, by comparing:

• a is the coefficient of x²: a=1/3

,

• b is the coefficient of x: b=-6

,

• c is the constant: c=7

[tex]a=\frac{1}{3},b=-6,c=7[/tex]

please can you help me to solve an equation ?Determine whether the statement is true or false : The slope of the line y = 1 is zero

Answers

The given line equation is y=1.

The general form of the line equation is

[tex]y=mx+b[/tex]

Comparing this equation with y=1, we get

[tex]mx+b=1[/tex]

[tex]mx+b=(0)x+1[/tex]

Compare coefficient of x and constant term, we get

[tex]m=0\text{ and b=1}[/tex]

Here m=0 is the slope of the line y=1.

Hence the slope of the line y = 1 is zero.

This statement is true.

​(d) Find the domain of function R. Choose the correct domain below.

Answers

Answer:

d

Step-by-step explanation:

The number of years must be non-negative.

This eliminates all of the options except for d.

Ganymede is one of Jupiter’s moons. It takes Ganymede 7 days, 3 hours, and 15 minutes to make one rotation around Jupiter. How many hours does it take Ganymede to complete a full rotation? Show your work using the correct conversion factors and make sure all conversion factors are labeled with appropriate units. You should have more than one conversion factor shown.

Answers

The number of hours it will take Ganymede to complete a full rotation is; 171.25 hours.

What is the fundamental principle of multiplication?

Multiplication is the mathematical operation that is used to determine the product of two or more numbers. If an event can occur in m different ways and if following it, a second event can occur in n different ways, then the two events in succession can occur in m × n different ways.

The first step must be to convert days to hours. A day is equivalent to the 24 hours.

1 day = 24 hours

24 x 7 = 168 hours

The second step must be to convert minutes to hours,

60 minutes = 1 hour

15 / 60 = 0.25 hours

Now, add up all the converted hours together;

Total hours = 168 hours + 3 hours + 0.25 hours

Total hours = 171.25 hours

Learn more about multiplications;

https://brainly.com/question/14059007

#SPJ1

I need help with this problem

Answers

Answer:

y = x-13

Step-by-step explanation:

So this question asks for two numbers. Let's call them x and y.

Since we know that x is larger than y, and the difference between them is 13, x must be greater than y by 13. So, x = y + 13
x = y + 13 isn't what the question is asking for though, so we have to isolate y to make one side contain only x and numbers. To do that, subtract both sides by 13 to get:
x - 13 = y

rearrange:

y = x - 13

A six -sided fair number cube is rolled once . What is the probability that the result is a 2 or a 5? Select one :1/61/32/55/6

Answers

total number of sides = 6

sides that are 2 or 5 = 2

Divide the sides thata re 2 or 5 (2) by the total sides:

2 /6 = 1/3

Find the area and perimeter of a rhombus with a diagonals 15 in and 20 in

Answers

Given data:

The diagonals of rhombus d1 = 15 in and d2 = 20 in

The area of the rhombus,

[tex]\begin{gathered} A=\frac{1}{2}\times d1\times d2 \\ A=\frac{1}{2}\times15\times20 \\ A=150\text{ inches sq.} \end{gathered}[/tex]

Now, to find the perimeter of rhombus we first find the side of rhombus.

The diagonal of the rhombus is considered as diameter, so the radius will be

d1 = r1 = 7.5

d2 = r2 = 10

So, by using the pythagorean theorem we find the side of the rhombus

[tex]\begin{gathered} h^2=(7.5)^2+(10)^2 \\ h^2=56.25+100 \\ h^2=156.25 \\ h=12.5 \end{gathered}[/tex]

The perimeter of the rhombus is,

[tex]\begin{gathered} P=4\times side \\ P=4\times12.5 \\ P=50\text{ inches} \end{gathered}[/tex]

Thus, the area is 150 in. sq. and perimeter is 50 in.

7/9 ÷ 3/9 = ????????

Answers

7/9 ÷ 3/9 =

= (7•9)/( 3•9)

= 63/27 = 7/3

Answer is 7/3

Evaluate.

1045+(834−612)

Enter your answer as a mixed number in simplest form by filling in the boxes.

Answers

The value of the expression given as 1045 + (834 − 612) is 1267

How to evaluate the expression?

The expression is given as

1045+(834−612)

Evaluate the brackets in the above expression

So, we have the following equation

1045 + (834 − 612) = 1045 + 222

Evaluate the sum in the above expression

So, we have the following equation

1045 + (834 − 612) = 1267

The above equation/expression cannot be solved further

So, we make our conclusion

Hence, the solution to the expression is 1045 + (834 − 612) = 1267

Read more about expressions at

https://brainly.com/question/4344214

#SPJ1

Complete the following truth table. Use T for true and F for false.You may add more columns, but those added columns will not be graded.

Answers

Given,

The equation of table is

[tex]\urcorner p\wedge\urcorner q^{}[/tex]

Here,

[tex]\begin{gathered} p\text{ q }\urcorner p\text{ }\urcorner q\text{ }\urcorner p\wedge\urcorner q \\ T\text{ T F F F} \\ T\text{ F F T F} \\ F\text{ T T F F} \\ F\text{ F T T T} \end{gathered}[/tex]

Hence, the output of the table is F, F, F, and T

A car rental company offers two plans for renting a car.



Plan A: 35 dollars per day and 10 cents per mile

Plan B: 55 dollars per day with free unlimited mileage



How many miles would you need to drive in one day for plan B to save you money?

Answers

Answer:

Less than 200 miles, A is cheaper

At 200 miles, they are the same

Over 200 miles, B is less expensive

Step-by-step explanation:

Suppose you are driving to your vacation destination. You are driving at an average rate of 60 miles per hour. You must drive a total of 235 miles. If you had already driven 55 miles, how long will it take you to reach your destination?

Answers

[tex]\begin{gathered} \text{speed}\Rightarrow60miles\text{ /hour} \\ \text{already driven=55 miles} \\ \text{total distance=235 miles} \\ \text{need to cover=235-55=180miles } \\ \text{time taken to complete the remaining distance is,} \\ t=\frac{180}{60}=3\text{ hour} \end{gathered}[/tex]


What is the volume of this container? 4 in.
4 in.
1 in.
4 in.
2 in.
64 in²
56 in³
56 in²
64 in³

Answers

An object's capacity is expressed in terms of volume.

The container's volume is 56 in3.

How to find the container's volume?

The capacity of an object is expressed in terms of volume.

A cup's capacity is said to be 100 ml, for instance, if it can hold 100 ml of water in its brim.

The quantity of space a three-dimensional item takes up can also be referred to as volume.

The figure's volume is equal to Bh.

V = Bh

where

B is the base's region.

h is the figure's height.

h = 4 in

The area of the base B is

B = (4*4) - (1*2) = 14 in

V = 14*4 = 56 inch cube.

To learn more about volume, refer

https://brainly.com/question/463363

#SPJ13

Solve. Richard Saltwood's car averages 270 miles on its 18-gallon tank of gas. If Richard runs out of gas and a mechanic comes to his rescue with 2 3/10 gallons of gas, determine how far his car can go. Round to the nearest mile.

Answers

ANSWER:

35 miles

STEP-BY-STEP EXPLANATION:

We can make a proportion, since we know the number of miles per 18 gallons, so for 2 3/10 gallons we can calculate the number of miles proportionally

[tex]\begin{gathered} 2\frac{3}{10}=\frac{23}{10} \\ \text{The proportion is} \\ \frac{270}{18}=\frac{x}{\frac{23}{10}}\rightarrow\frac{270}{18}=\frac{10x}{23} \end{gathered}[/tex]

solving for x:

[tex]\begin{gathered} 18\cdot10x=270\cdot23 \\ 180x=6210 \\ x=\frac{6210}{180} \\ x=34.5\cong35 \end{gathered}[/tex]

Therefore you can travel a total of 35 miles.

There area 17,60 gaffs in a mile . What is the number of yards in a mile rounded to the nearest hundred? 1- How many places are in this number?2- What are the places? 3- What place are you rounding to?4- What digit is in that place?5- What place is to the right of the place you are rounding to?6- What digit is in that place?Is this digit 5 or greater?7- What is the rule for digits 5 or greater? 8- Will you need to change the hundreds digit? If so, to what? 9- Which places will you change to 0s?10- What is the number of yards in a mile rounded to the nearest hundred?

Answers

itWe were given the following:

[tex]1760yard=1mi[/tex]

1.

There are 4 places in this number

2.

1 thousand + 7 hundred + 6 tens + 0 units

3.

We are rounding to the hundreds place

4.

Seven (7)

5.

To the right, we have the ''tens''

6.

Six(6). Yes, the digit is 5 or greater

7.

Digits that are 5 or greater than are approximated to the next digit

8.

Yes. From 7 to 8

9.

The tens place will be changed to 0

10.

The number of yards in a mile rounded to the nearest hundred is:

[tex]\begin{gathered} 1,760yards=1mi \\ 1,760\approx1800 \\ =1,800yards(to\text{ the nearest hundred)} \end{gathered}[/tex]

anwser? please i need this to pass my test and i dont understand

Answers

Based on the conversion rate between kilometers and miles, the data to complete the table is:

8 miles : 12.88 kilometers 0.93 miles  : 1.5 kilometers 5.58 miles  : 9 kilometers 3.1 miles : 4.991 kilometers

How to convert miles to kilometers?

For every given mile, there will be 1.61 kilometers. To convert miles to kilometers therefore, you have to multiply the number of miles by the number of kilometers in a mile.

If given 8 miles, the number of kilometers is:

= 8 x 1.61

= 12.88 kilometers

If given 3.1 miles, the number of kilometers is:

= 3.1 x 1.61

= 4.991 kilometers

If given 1.5 kilometers, then the number in miles is:

= 1.5 x 0.62

= 0.93 miles

If given 9 kilometers, then the number in miles is:

= 9 x 0.62

= 5.58 miles

Find out more on miles and kilometers at https://brainly.com/question/25756854

#SPJ1

Factor the expression 54x2-6y^2?

Answers

[tex]\begin{gathered} 54x^2-6y^2 \\ 9\cdot\: 6x^2-6y^2 \\ 6\mleft(9x^2-y^2\mright) \\ 6\mleft(3x+y\mright)\mleft(3x-y\mright) \end{gathered}[/tex]

Other Questions
How do I draw the fourth diagram in a square? How do I know how many sticks in the sixth diagram? How many thumb tacks are in the seventh diagram? Use the data shown to answer the questions that follow.Person Minutes Shopping in the Store Amount Spent on Groceries at Checkout1 56.5 177.52 84.4 260.23 66.3 197.94 47.3 82.95 48.7 147.16 47.1 148.37 60.7 182.18 76 228Which graph is most appropriate for these data?Bar ChartScatter PlotPie ChartTime Series PlotCopy and paste the above data into Excel and make the most appropriate graph for these data. Make sure the x-axis represents the "Minutes Shopping in the Store" and that the y-axis represents the "Amount Spent on Groceries at Checkout."I have successfully created this graph in Excel as requested.I have not graphed the data.One of the values in the plot of this data does not fit the pattern of the other values. It would be called an outlier.What is the x-value and y-value of the outlier?( Number , Number )Hint: If you hover your mouse over the points in your Excel graph it will show you the exact x-value and y-value of the point. How do people react to a crisis they are going through in primary and secondary group? 6ft , 5ft , c. What is the length of the hypotenuse Convert 3.5 104 to standard form. (5 points)3,50035,0000.000035 Torrie has $430 and Sonia has $760. John has the same number of dollars more than Torrie as he has less than Sonia.How much money does John have? a client is receiving levofloxacin for treatment of urinary tract infection. which finding warrants an immediate call to the primary health care provider (phcp)? Alcohol is considered a gateway drug because how is a circle square and triangle related? What happens when the cell of a unicellular organism dies? Why was it important to have endothermic and exothermic processes in the building materials of your train? mcdonald's claims that their monopoly game has a win rate of 25%. if you play the game 500 times, what is the probability that your win rate will be between 21% and 23%? Mara: Tengo que comprar un cinturn. Prefiero algo hecho a mano y tnico.Ral: Tienes que ir _____________________. A. al mall B. al supermercado C. a la feria artesanal D. al almacn The Federal form 1040 is used by whom to file taxes?O IndividualsO CompaniesO Law EnforcementO Nonprofit Organizations N2.Find the area of each sector. Round your answer to the nearest tenth.1.sector PQR29Rsector JKL8 cm 135K6mP90LArea =Area =1 0.72 1.6mcm3 28.74 75.4 Write an inequality that represents the situation.A certain swimming pool holds no more than 700,000 gallons. How many gallons might be in the pool? Angel and his children went into a restaurant and will buy hotdogs and drinks. Eachhotdog costs $5 and each drink costs $1.50. Angel has a total of $50 to spend onhotdogs and drinks. Write an inequality that would represent the possible values forthe number of hotdogs purchased, h, and the number of drinks purchased, d. what prevents coral reefs from surviving below the euphotic zone? group of answer choices high water density high pressure inadequate sunlight saline water damage from storms Question 1 of 10What is the slope of the line shown below?A. 3B. -1/3C. -3D. 1/3 When she was transferred to Miami, she had to buy a new car,find a nice apartment, andopening a checking account.